LSAT and Law School Admissions Forum

Get expert LSAT preparation and law school admissions advice from PowerScore Test Preparation.

 Administrator
PowerScore Staff
  • PowerScore Staff
  • Posts: 8915
  • Joined: Feb 02, 2011
|
#36970
Complete Question Explanation

Justify the Conclusion—Formal Logic. The correct answer choice is (C)

In this stimulus, the political theorist presents a series of conditional statements, which can be
diagrammed as follows:

For an alliance to be strong, it must respond aggressively to problems:
  • AS :arrow: RA
An alliance will respond aggressively only if every member perceived gravity:
  • RA :arrow: PG
But the EU countries will not perceive a problem to be grave unless they all agree that it threatens their
alliance’s economy:
  • PG :arrow: ATAE

    What we then have is the following chain of logic:
    AS :arrow: RA :arrow: PG :arrow: ATAE

    We can also draw from these statements the following contrapositive chain:
    ATAE :arrow: PG :arrow: RA :arrow: AS
The theorist concludes that, with respect to the European Union, not all members of the will be strong
in foreign policy (this is the condition ‘AS’ diagrammed above). We are then asked to justify this
conclusion. This means that from the answer choices we should select the one which, when added to the
premises in the stimulus, allows for this conclusion (AS) to be properly drawn.

Answer choice (A): This answer choice concerns an irrelevant group—those countries which refuse to
join an alliance. Since this information does not affect the conditional reasoning in the stimulus, this
answer choice is incorrect.

Answer choice (B): This choice supplies another condition, and relates to the aggression level of
individual countries rather than the responses of alliances.

Answer choice (C): This is the correct answer choice. If problems that appear economically
threatening to some countries do not appear so to others, then at least some countries do not perceive the
gravity of the threat, meaning PG (that is, not all countries in the alliance perceive the threat), and if we
know this then we know PG :arrow: RA :arrow: AS. In other words, from this information we
can logically conclude AS.

Answer choice (D): It does not matter which countries are weak, and this choice does not prove that any
countries do fail to perceive economic relevance, so this choice does not trigger any of the conditions in
the stimulus needed to justify the conclusion.

Answer choice (E): The benefits derived by individual countries from alliance membership are irrelevant
to the conclusion we seek to justify.

Get the most out of your LSAT Prep Plus subscription.

Analyze and track your performance with our Testing and Analytics Package.